2019-2020/TES/DS/DS_19_10_14/DS_19_10_14.tex

192 lines
8.0 KiB
TeX
Raw Blame History

This file contains ambiguous Unicode characters

This file contains Unicode characters that might be confused with other characters. If you think that this is intentional, you can safely ignore this warning. Use the Escape button to reveal them.

\documentclass[a4paper,10pt]{article}
\usepackage{myXsim}
%\usepackage[inline]{enumitem}
\usepackage{tasks}
\title{DS 2}
\tribe{Terminale ES-L}
\date{14 octobre 2019}
\begin{document}
\maketitle
Le barème est donné à titre indicatif, il pourra être modifié.
Une part importante de la note sera dédiée à la rédaction, aux explications et à l'utilisation des notations mathématiques.
\begin{exercise}[subtitle={QCM},points=4]
\textit{Cet exercice est un QCM (questionnaire à choix multiples).}
\textit{Pour chacune des questions posées, une seule des trois ou quatre réponses est exacte. Recopier le numéro de la
question et la réponse exacte. Aucune justification nest demandée. Une réponse exacte rapporte 1 point,
une réponse fausse ou labsence de réponse ne rapporte ni nenlève de point. Une réponse multiple ne
rapporte aucun point.}
\begin{enumerate}
\item Dans une ville de la région, sur une population de \np{4200}~ habitants, $36$\,\% ont pris connaissance de la publicité lors de la première semaine de la campagne. Le nombre d'habitants de cette ville ayant pris connaissance de la publicité lors de la première semaine de la campagne est:
\begin{tasks}(4)
\task \np{2688}
\task \np{1512}
\task \np{1167}
\task \np{4164}
\end{tasks}
\item Dans une concession automobile de la région, le temps d'attente, exprimé en minutes, avant d'être reçu par un conseiller commercial peut être modélisé par une variable aléatoire qui suit la loi uniforme sur l'intervalle [1~;~10].
Un visiteur se présente. Quelle est la probabilité qu'il attende au moins 5 minutes avant d'être reçu par un conseiller commercial ?
\begin{tasks}(4)
\task 0,4
\task 0,5
\task $\dfrac{4}{9}$
\task $\dfrac{5}{9}$
\end{tasks}
\item Entre 2006 et 2018, dans un restaurant universitaire, le prix d'un repas est passé de 2 euros à 3,50~euros en augmentant chaque année de x\,\%. Parmi ces valeurs, la valeur la plus proche de $x$ est:
\begin{tasks}(4)
\task 6,25
\task 4,77
\task 14,58
\task0,85
\end{tasks}
\item On donne $u_{n+1} = 0.9u_n+\np{24000}$ et $v_n = u_n-\np{240000}$. Alors
\begin{multicols}{2}
\begin{enumerate}
\item $(v_n)$ est arithmétiques de raison \np{24000}
\item $(v_n)$ est arithmétiques de raison \np{-240000}
\item $(v_n)$ est géométrique de raison \np{0.9}
\item $(v_n)$ est ni géométrique ni arithmétiques.
\end{enumerate}
\end{multicols}
\end{enumerate}
\end{exercise}
\begin{exercise}[subtitle={Population}, points=4]
Une ville a vu sa population augmenter de \np{124000} habitants début 2014 à \np{128460} habitants début 2015.
\begin{enumerate}
\item Calculer le taux d'évolution de la population de cette ville entre 2014 et 2014.
\item Proposer un modèle pour calculer le nombre d'habitants de cette ville si l'on suppose que ce taux d'évolution reste constant.
\item En comparant avec l'évolution de villes analogues, les démographes pendent que la ville comptera \np{198600} habitants début 2030. Cette estimation est-elle compatible avec le modèle construit à la question 2?
\end{enumerate}
\end{exercise}
\begin{exercise}[subtitle={Grand seigneur}, points=4]
Dans le restaurant qu'il fréquente chaque midi, Max dépense entre 10 et 35euros, "tout compris" selon ses choix. On modélise le prix d'un repas de Maw par la variable aléatoire $X$ dont la loi est uniforme entre 10 et 35\euro.
\begin{enumerate}
\item Calculer la probabilité que Max dépense moins de 17,50\euro.
\item Calculer $P(19 < X < 27)$
\item Combien va-t-il payer en moyenne son repas?
\end{enumerate}
Max, qui paye en espèces et se s'embarrasse pas avec la monnaie, a pour habitude de donner au serveur un somme arrondie au multiple de 5 supérieur.
\begin{enumerate}
\setcounter{enumi}{3}
\item Quel montant donne-t-il quand son repas coûte 19\euro?
\end{enumerate}
On modélise la somme donnée par Max pour payer son repas par la variable aléatoire $Y$.
\begin{enumerate}
\setcounter{enumi}{4}
\item Quelles valeurs peut prendre $Y$?
\item Est-ce que $Y$ suit une loi uniforme? Si oui, préciser les paramètres. Si non, quelle est sa loi?
\end{enumerate}
\end{exercise}
\begin{exercise}[subtitle={Occupation des lits}, points=8]
On donne ci-dessous la courbe $\mathcal{C}$ représentative d'une fonction $f$ définie et dérivable sur l'intervalle $\intFF{0}{4}$. On note $f'$ la fonction dérivée de $f$.
La droite $\mathcal{D}$ est la tangente à la courbe $\mathcal{C}$ au point d'abscisse $0$ ; elle passe par le point A de coordonnées (0,5~;~1).
La tangente $T$ à la courbe $\mathcal{C}$ au point d'abscisse 1.45 est parallèle à l'axe des abscisses.
\begin{minipage}{0.5\textwidth}
\def\tkzRatioLineGrid{0.75}
\begin{tikzpicture}[yscale=3, xscale=2, baseline=(a.north)]
\tkzInit[xmin=0,xmax=4,xstep=1,
ymin=0,ymax=1.5,ystep=1]
\tkzGrid[sub, ligne width=1.5]
\tkzAxeXY[up space=0.2,right space=0.2]
\tkzFct[domain=0:4,color=black,very thick]%
{0.1*x*(x-4)*(x-5)}
\draw (3.6,0.4) node {$\mathcal{C}$};
\tkzFct[domain=0:4,color=black,thick]%
{2*x}
\draw (0.5,1) node {$\times$} node[above left] {A};
\draw [<->, thick] (1,1.31) -- (1.46,1.31) node {$\times$} node[above left] {$T$} -- (2, 1.31);
\tkzFct[domain=2.3:3.7,color=black,thick]%
{-0.7*x+2.7}
\draw (3,0.6) node[very thick] {\large +};
\end{tikzpicture}
\end{minipage}
\begin{minipage}{0.5\textwidth}
\textbf{Partie A}
\medskip
Dans cette partie les réponses seront obtenues par lecture graphique.
\medskip
\begin{enumerate}
\item Donner la valeur de $f'(1.45)$.
\item Déterminer une équation de la droite $\mathcal{D}$.
\item Proposer un intervalle sur lequel la fonction semble concave.
\end{enumerate}
\end{minipage}
\bigskip
\textbf{Partie B}
\medskip
La fonction $f$ est définie sur l'intervalle [0~;~4] par $f(x) = 0.3x^3-0.9x^2+2x$
On admet que son tableau de variation est
\begin{center}
\begin{tikzpicture}[baseline=(a.north)]
\tkzTabInit[lgt=1,espcl=2]{$x$/1,$f(x)$/1}{0, 1.46, 4}
\tkzTabVar{-/ 0, +/ 1.31, -/ 0}
\end{tikzpicture}
\end{center}
\begin{enumerate}
\item Démontrer que l'équation $f(x) = 1.1$ admet une unique solution sur $\intFF{0}{1}$.
\item Donner une valeur approchée à $10^{-2}$ de cette solution.
\end{enumerate}
\bigskip
\textbf{Partie C}
\medskip
En Europe, les observateurs d'une maladie nécessitant une hospitalisation considèrent qu'ils peuvent modéliser par cette fonction $f$ l'évolution du nombre de lits occupés par des malades pendant les quatre mois d'hiver.
Pour tout $x$ appartenant à l'intervalle [0~;~4], $f(x)$ représente le nombre de lits occupés, exprimé en million, à l'instant $x$, exprimé en mois.
Un journal affirme que cet hiver:
{\setlength\parindent{10mm}
\begin{itemize}
\item le nombre de lits occupés a dépassé le million pendant plus d'un mois.
\item le maximum de lits occupés a été atteint autour du milieu du deuxième moi.
\item après 3 mois, la diminution du nombre de lit a ralentit.
\end{itemize}}
Que dire de ces trois affirmations ?
\end{exercise}
\end{document}
%%% Local Variables:
%%% mode: latex
%%% TeX-master: "master"
%%% End: